Proof of Monotone Classes and Subsets in Measure Theory

  • Thread starter Thread starter fishturtle1
  • Start date Start date
  • Tags Tags
    Classes Proof
Click For Summary
SUMMARY

The discussion focuses on proving that the collection of sets ##\mathcal{F}## is a subset of the monotone classes ##\sum## and ##\sum'## in measure theory. The proof establishes that both ##\sum## and ##\sum'## are monotone classes by demonstrating that they satisfy the properties of monotonicity for increasing and decreasing sequences of sets. Furthermore, it is shown that ##\mathcal{F}## is stable under intersection and is included in the sigma-algebra generated by ##\mathcal{F}##, leading to the conclusion that ##\mathcal{F} \subseteq \sum \subseteq \sum'##, thereby completing the proof.

PREREQUISITES
  • Understanding of measure theory concepts, particularly monotone classes.
  • Familiarity with sigma-algebras and their properties.
  • Knowledge of set operations, including unions and intersections.
  • Basic proficiency in mathematical notation and logic.
NEXT STEPS
  • Study the properties of monotone classes in detail.
  • Explore the relationship between sigma-algebras and measure spaces.
  • Learn about the implications of the Monotone Convergence Theorem in measure theory.
  • Investigate examples of monotone classes in various measure-theoretic contexts.
USEFUL FOR

Mathematicians, students of advanced calculus, and researchers in measure theory who are looking to deepen their understanding of monotone classes and their applications in probability and analysis.

fishturtle1
Messages
393
Reaction score
82
Homework Statement
A family ##\mathcal{M} \subset \mathcal{P}(X)## which contains ##X## and is stable under countable unions of increasing sets and countable intersections of decreasing sets
$$(A_n)_{n\in\mathbb{N}}, A_1 \subset A_2 \subset \dots \subset A_n \subset A_{n+1} \uparrow A = \bigcup_{n=1}^{\infty} A_n \Rightarrow A\in \mathcal{M}$$
$$(B_n)_{n\in\mathbb{N}}, B_1 \supset B_2 \supset \dots \supset B_n \supset B_{n+1} \downarrow B = \bigcap_{n=1}^{\infty}B_n \Rightarrow B \in \mathcal{M}$$
is called a ##\textit{monotone class}##. Assume that ##\mathcal{M}## is a monotone class and ##\mathcal{F} \subset \mathcal{P}(X)## any family of sets.

(iii) If ##\mathcal{F}## is ##\cap##-stable, i.e. ##F, G \in \mathcal{F} \Rightarrow F \cap G \in \mathcal{F}##, then so is ##m(\mathcal{F})##.
[Hint: show that the families
$$\sum := \lbrace M \in m(\mathcal{F}) : M \cap F \in m(\mathcal{F}), \forall F \in \mathcal{F} \rbrace$$
$$\sum' := \lbrace M \in m(\mathcal{F}) : M \cap N \in m(\mathcal{F}), \forall N \in m(\mathcal{F}) \rbrace$$
are again monotone classes satisfying ##\mathcal{F} \in \sum, \sum'##.
Relevant Equations
Here ##m(\mathcal{F})## is the smallest monotone class that contains ##\mathcal{F}##. In other words, if ##L## is a monotone class containing ##\mathcal{F}##, then ##m(\mathcal{F}) \subseteq L##.
My question is how to show ##\mathcal{F} \subset \sum'##. Here is my work for the problem:

Proof of hint: First we'll show ##\sum## is a monotone class. Let ##(A_n)_{n\in\mathbb{N}} \subset \sum## and ##F \in \mathcal{F}##. There are two things to verify. Suppose ##(A_n) \uparrow A = \bigcup_{n=1}^{\infty} A_n##. Then ##(A_n \cap F) \uparrow (A \cap F)##. But ##A_n \cap F \in m(\mathcal{F})## for all ##n##. Hence, ##A \cap F \in m(\mathcal{F})## for all ##F \in \mathcal{F}##. Thus, ##A \in \sum##.

Now, suppose ##(A_n) \downarrow A = \bigcap_{n=1}^{\infty} A_n##. Then ##(A_n \cap F) \downarrow (A \cap F)##. Since ##A_n \cap F \in m(\mathcal{F})## for all ##n##, we have ##A \cap F \in m(\mathcal{F})## for all ##F \in \mathcal{F}##. Thus, ##A \in \sum##. We may conclude ##\sum## is a montone class.

By a similar argument, we can show ##\sum'## is a monotone class. Next, we'd like to show ##\mathcal{F} \subset \sum, \sum'##. Since ##\mathcal{F}## is stable under intersection and ##\mathcal{F} \subset m(\mathcal{F})##, we have ##\mathcal{F} \subset \sum##. I think I need to show ##\sum \subset \sum'## which would give me ##\mathcal{F} \subset m(\mathcal{F}) \subset \sum \subset \sum'##. And clearly ##\sum' \subset m(\mathcal{F})##. This would give ##\sum' = m(\mathcal{F})##, completing the proof.

I tried: Suppose ##A \in \sum##. Let ##N \in m(\mathcal{F})##. We'd like to show ##A \cap N \in m(\mathcal{F})##. If ##N \in \mathcal{F}##, then this is true. Otherwise...
 
  • Like
Likes member 587159
Physics news on Phys.org
You have shown that ##\mathcal{F} \subseteq \sum## and ##\sum## is a monotone class. Hence, ##m(\mathcal{F}) \subseteq \sum##.

This allows us to show ##\mathcal{F} \subseteq \sum'##:

If ##A \in \mathcal{F}## and ##N \in m(\mathcal{F})##, then we must show that ##A \cap N \in m(\mathcal{F})##. But now we know that ##N \in \sum##, so since ##A \in \mathcal{F}## we have ##A \cap N \in m(\mathcal{F})##. We conclude ##A \in \sum'##.
 
  • Informative
Likes fishturtle1
Math_QED said:
You have shown that ##\mathcal{F} \subseteq \sum## and ##\sum## is a monotone class. Hence, ##m(\mathcal{F}) \subseteq \sum##.

This allows us to show ##\mathcal{F} \subseteq \sum'##:

If ##A \in \mathcal{F}## and ##N \in m(\mathcal{F})##, then we must show that ##A \cap N \in m(\mathcal{F})##. But now we know that ##N \in \sum##, so since ##A \in \mathcal{F}## we have ##A \cap N \in m(\mathcal{F})##. We conclude ##A \in \sum'##.
That makes sense, thank you!
 
  • Like
Likes member 587159
Question: A clock's minute hand has length 4 and its hour hand has length 3. What is the distance between the tips at the moment when it is increasing most rapidly?(Putnam Exam Question) Answer: Making assumption that both the hands moves at constant angular velocities, the answer is ## \sqrt{7} .## But don't you think this assumption is somewhat doubtful and wrong?

Similar threads

  • · Replies 8 ·
Replies
8
Views
2K
Replies
2
Views
2K
  • · Replies 5 ·
Replies
5
Views
3K
  • · Replies 13 ·
Replies
13
Views
2K
  • · Replies 1 ·
Replies
1
Views
2K
Replies
2
Views
2K
Replies
1
Views
2K
  • · Replies 2 ·
Replies
2
Views
2K
  • · Replies 2 ·
Replies
2
Views
2K
  • · Replies 1 ·
Replies
1
Views
2K